SchoolWork-LaTeX/随机过程/平时作业/第九周作业.tex
423A35C7 5906ac1efc 重构目录层次
0-课程笔记
1-平时作业
2-实验报告
3-期末大作业
2024-09-02 18:32:58 +08:00

290 lines
12 KiB
TeX
Raw Permalink Blame History

This file contains ambiguous Unicode characters

This file contains Unicode characters that might be confused with other characters. If you think that this is intentional, you can safely ignore this warning. Use the Escape button to reveal them.

\documentclass[全部作业]{subfiles}
\input{mysubpreamble}
\begin{document}
\setcounter{chapter}{3}
\setcounter{section}{2}
\section{练习题\thesection}
\begin{enumerate}
\questionandanswerSolution[1]{
$X$的状态空间为$\{ 1,2,3 \}$,转移概率矩阵为$\begin{pmatrix}
0 & \frac{1}{2} & \frac{1}{2} \\
\frac{1}{2} & 0 & \frac{1}{2} \\
0 & \frac{1}{3} & \frac{2}{3} \\
\end{pmatrix}$,对任意$n\geqslant 1$,求$f_{1,2}^{(n)}$
}{
$$
\begin{cases}
F_{12}(u)=p_{12}u+p_{11}uF_{12}(u)+p_{13}uF_{32}(u)\quad \mycircle{1} \\
F_{22}(u)=p_{22}u+p_{21}uF_{12}(u)+p_{23}uF_{32}(u) \quad \mycircle{2} \\
F_{32}(u)=p_{32}u+p_{31}uF_{12}(u)+p_{33}uF_{32}(u) \quad \mycircle{3} \\
\end{cases}
$$
\mycircle{1}\mycircle{3}可得
$$
\begin{cases}
F_{12}(u)=\frac{1}{2}u+\frac{1}{2}uF_{32}(u) \\
F_{32}(u)=\frac{1}{2}u+\frac{2}{3}uF_{32}(u) \\
\end{cases}
$$
解得
$$
\begin{cases}
F_{32}(u)=\frac{\frac{1}{2}u}{1-\frac{2}{3}u} \\
F_{12}(u)=\frac{1}{2}u+\frac{\frac{1}{4}u^{2}}{1-\frac{2}{3}u} \\
\end{cases}
$$
所以
$$
F_{12}(u)=\frac{1}{2}u+\sum_{n=1}^{\infty} \frac{1}{4}u^{2}\left( \frac{2}{3}u \right) ^{n-1} = \frac{1}{2}u+\sum_{n=1}^{\infty} \frac{1}{4} \left( \frac{2}{3} \right) ^{n-1} u^{n+1} = \frac{1}{2}u+ \sum_{n=2}^{\infty} \frac{1}{2} \left( \frac{2}{3} \right) ^{n-2} u^{n}
$$
综上,$\displaystyle f_{12}^{(1)}=\frac{1}{2}u, \quad f_{12}^{(n)}=\frac{1}{2}\left( \frac{2}{3} \right) ^{n-2}(n\geqslant 2)$
}
\questionandanswerSolution[2]{
一个盗窃犯长期在$A=1,B=2,C=3$三地流传作案,治安部门调查后发现他每年作案次数服从强度为$3$的泊松分布,而连续两次作案的地点变化服从转移概率矩阵$\begin{pmatrix}
0 & 0 & 1 \\
\frac{1}{2} & \frac{1}{2} & 0 \\
0 & 1 & 0 \\
\end{pmatrix}$,并且作案次数与作案地点无关。已知此人刚刚在$A$地作案,试估计一年内他在$A$地再作案的概率。
}{
由题意可知,作案地点的变化为时齐马氏链,从而所求的概率为$\displaystyle E_{k\sim \operatorname{Poi}(3)}f_{11}^{(k)}$
% 这里有个问题,如果$p_{21}=1,p_{22}=0$的话,那方程组中就没有$\frac{1}{2}u$这一项,那么$F_{21}(u)=0$
$$
\begin{cases}
F_{11}(u)=uF_{31}(u) & \mycircle{1} \\
F_{21}(u)=\frac{1}{2}uF_{11}(u)+\frac{1}{2}u &\mycircle{2} \\
F_{31}(u)=uF_{21}(u) &\mycircle{3} \\
\end{cases}
$$
\mycircle{1}\mycircle{3}$F_{11}(u)=u^{2}F_{21}(u)$,代入\mycircle{2}可得$F_{21}(u)=\frac{1}{2}u^{3}F_{21}(u)+\frac{1}{2}u$,解得$\displaystyle F_{21}(u)=\frac{\frac{1}{2}u}{1-\frac{1}{2}u^{3}}$,所以
$$
F_{11}(u)=\frac{\frac{1}{2}u^{3}}{1-\frac{1}{2}u^{3}}=\sum_{n=1}^{\infty} \frac{1}{2}u^{3} \left( \frac{1}{2}u^{3} \right) ^{n-1}=\sum_{n=1}^{\infty} \left( \frac{1}{2} \right) ^{n} u^{3n}
$$
$k=3n$,则$f_{11}^{(k)}=\begin{cases}
\left( \frac{1}{2} \right) ^{\frac{k}{3}},\quad & k=3n \\
0,\quad & k\neq 3n \\
\end{cases}\ ,n\in \mathbb{Z}^{+}, k\in \mathbb{Z}^{+}$,所以
$$
E_{k\sim \operatorname{Poi}(3)}f_{11}^{(k)}=\sum_{n=1}^{\infty} \left( \frac{1}{2} \right) ^{n} \cdot \frac{3^{3n}}{(3n)!}e^{-3} = \sum_{n=1}^{\infty} \frac{\left( 3 \sqrt[3]{\frac{1}{2}} \right) ^{3n}}{(3n)!} e^{-3}=e^{3 \sqrt[3]{\frac{1}{2}}} e^{-3} = e^{\frac{3}{\sqrt[3]{2}}-3}
$$
}
\questionandanswerSolution[3]{
$X$的状态空间为$\{ 1,2,3 \}$,转移概率矩阵为$\begin{pmatrix}
\frac{1}{2} & \frac{1}{2} & 0 \\
\frac{2}{3} & \frac{1}{3} & 0 \\
0 & \frac{1}{3} & \frac{2}{3} \\
\end{pmatrix}$。求$f_{3,2}$$m_{3,2}$
}{
$$
\begin{cases}
f_{32}=\frac{1}{3}+\frac{2}{3}f_{32},\quad & \mycircle{1} \\
f_{12}=\frac{1}{2}+\frac{1}{2}f_{12},\quad & \mycircle{2} \\
f_{22}=\frac{1}{3}+\frac{2}{3}f_{12},\quad & \mycircle{3} \\
\end{cases}\qquad \begin{cases}
m_{32}=f_{32}+\frac{2}{3}m_{32},\quad & \mycircle{4} \\
m_{12}=f_{12}+\frac{1}{2}m_{22},\quad & \mycircle{5} \\
m_{22}=f_{22}+\frac{2}{3}m_{12},\quad & \mycircle{6} \\
\end{cases}
$$
\mycircle{1}可得$f_{32}=1$,代入\mycircle{4}可得$m_{32}=3$
}
\questionandanswerSolution[4]{
若一篇文稿有$n$个错误,每次校阅至少能发现一个,但留下来的错误数在$0$$n-1$之间等可能存在.设原稿共有$a$个错误,问为了改正全部错误平均需要校阅几次?
}{
设随机过程$X$表示在某个时刻留下来的错误数,则$X$是时齐马氏链,状态空间为$\{1, 2, \cdots ,n-1\}$$X$的转移概率矩阵为
$$
\begin{pmatrix}
1 & 0 & 0 & \cdots & 0 \\
1 & 0 & 0 & \cdots & 0 \\
\frac{1}{2} & \frac{1}{2} & 0 & \cdots & 0 \\
\vdots & \vdots & \vdots & \ddots & \vdots \\
\frac{1}{n} & \frac{1}{n} & \frac{1}{n} & \cdots & 0 \\
\end{pmatrix}_{n\times n}
$$
所以
$$
\begin{cases}
f_{10}=1 \\
f_{20}=\frac{1}{2}+\frac{1}{2}f_{10}=1 \\
f_{30}=\frac{1}{3}+\frac{1}{3}f_{10}+\frac{1}{3}f_{20}=1 \\
\cdots \\
f_{n0}=\frac{1}{n}+\sum_{i=1}^{n} f_{i0}=1 \\
\end{cases} \quad
\begin{cases}
m_{10}=f_{10} \\
m_{20}=f_{20}+\frac{1}{2}m_{10}=1+\frac{1}{2}\times 1=\frac{3}{2} \\
m_{30}=f_{30}+\frac{1}{3}m_{10}+\frac{1}{3}m_{20}=1+\frac{1}{3}\times 1+\frac{1}{3}\times \frac{3}{2}=\frac{11}{6} \\
\cdots \\
m_{n0}=f_{n0}+\sum_{i=1}^{n-1} \frac{1}{n} m_{i0} \\
\end{cases}
$$
根据递推公式算出$m_{n0}$(不会算通项公式了),将$a$代入,则可知为了改正全部错误平均需要校阅$m_{a0}$次。
}
\questionandanswerSolution[6]{
Ehrenfest 模型)设一个坛子内装有红白两色共$N$个球,每次随机地从坛子中抽出一个球,把它换成另一种颜色后放回。以$X_n$表示经$n$次抽放后坛中的红球数,那么$X=\{ X_n\ ;\ n\geqslant 0 \}$为时齐马氏链。若开始时坛内只有一个红球,问平均要抽放多少次才能使坛内全是白球?
}{
可以观察到$X$类似带反射壁的随机游动,但向中间收敛的概率更大,转移概率矩阵为
$$
\begin{pmatrix}
0 & 1 & 0 & \cdots & 0 & 0 \\
\frac{1}{N} & 0 & \frac{N-1}{N} & \cdots & 0 & 0 \\
0 & \frac{2}{N} & 0 & \cdots & 0 & 0 \\
\vdots & \vdots & \vdots & \ddots & \vdots & \vdots \\
0 & 0 & 0 & \cdots & 0 & \frac{1}{N} \\
0 & 0 & 0 & \cdots & 1 & 0 \\
\end{pmatrix}
$$
开始时坛内只有一个红球,那么$X_0=1$,使坛内全是白球即$X_{?}=0$,所以所求的平均抽放次数为$m_{10}$
由于这是元素有限的本质类,从而一定是常返类,所以$\forall i,j \in \{ 1,2, \cdots N \}, f_{ij}=1$,所以可以直接列出$m_{ij}$的方程组。
$$
\begin{cases}
m_{00}&=1+m_{10} \\
m_{10}&=1+\frac{N-1}{N}m_{20}+\frac{1}{N}m_{00} \\
m_{20}&=1+\frac{2}{N}m_{10}+\frac{N-2}{N}m_{30} \\
\cdots \\
m_{n-1,0}&=1+\frac{N-1}{N}m_{n-2,0}+\frac{1}{N}m_{n0} \\
m_{n0}&=1+m_{n-1,0} \\
\end{cases}
$$
写成矩阵的形式为
$$
\begin{bmatrix}
1 & -1 & 0 & \cdots & 0 & 0 \\
-\frac{1}{N} & 1 & -\frac{N-1}{N} & \cdots & 0 & 0 \\
0 & -\frac{2}{N} & 1 & \cdots & 0 & 0 \\
\vdots & \vdots & \vdots & \ddots & \vdots & \vdots \\
0 & 0 & 0 & \cdots & 1 & -\frac{1}{N} \\
0 & 0 & 0 & \cdots & -1 & 1 \\
\end{bmatrix}
\begin{bmatrix}
m_{00} \\
m_{10} \\
m_{20} \\
\vdots \\
m_{n-1,0} \\
m_{n0} \\
\end{bmatrix}
= \begin{bmatrix}
1 \\
1 \\
1 \\
\vdots \\
1 \\
1 \\
\end{bmatrix}
$$
解得$m_{10}=\left( \begin{bmatrix}
1 & -1 & \cdots & 0 \\
-\frac{1}{N} & 1 & \cdots & 0 \\
\vdots & \vdots & \ddots & \vdots \\
0 & 0 & \cdots & 1 \\
\end{bmatrix} ^{-1} \begin{bmatrix}
1 \\
1 \\
\vdots \\
1 \\
\end{bmatrix}\right)(1, 0)$即为使坛内全是白球的平均抽放次数。
}
\begin{shaded}
\item[\textbf{补}]
(“赌徒输光模型”)考虑$\{ 0,1, \cdots ,N \}(N\geqslant 2)$上的$(q,p)$-简单随机游动($0<p,q<1,p+q=1$),其中两个端点是“完全吸收壁”(即$p_{00}=p_{NN}=1$。对$i \in \{ 1, \cdots ,N-1 \}$,计算$f_{i0}$(即初始状态为$i$时的“输光”概率)。
\end{shaded}
{
\begin{solution}\kaishu
由题意可知转移概率矩阵为
$$
\begin{bmatrix}
1 & 0 & 0 & \cdots & 0 & 0 \\
q & 0 & p & \cdots & 0 & 0 \\
0 & q & 0 & \cdots & 0 & 0 \\
\vdots & \vdots & \vdots & \ddots & \vdots & \vdots \\
0 & 0 & 0 & \cdots & 0 & p \\
0 & 0 & 0 & \cdots & 0 & 1 \\
\end{bmatrix}_{(N+1)\times (N+1)}
$$
则可以列出关于$f_{i0}$的方程组
$$
\begin{cases}
f_{00}=1 \\
f_{10}=qf_{00}+pf_{20} \\
f_{20}=qf_{10}+pf_{30} \\
\cdots \\
f_{n-2,0}=qf_{n-3,0}+pf_{n-1,0} \\
f_{n-1,0}=qf_{n-2,0}+pf_{n0} \\
f_{n_0}=f_{n_0} & \text{(这一条方程可以去除,因为$f_{n0}=0$}\\
\end{cases}
$$
写成矩阵的形式为
$$
\begin{bmatrix}
1 & 0 & 0 & \cdots & 0 & 0 \\
q & -1 & p & \cdots & 0 & 0 \\
0 & q & -1 & \cdots & 0 & 0 \\
\vdots & \vdots & \vdots & \ddots & \vdots & \vdots \\
0 & 0 & 0 & \cdots & -1 & p \\
0 & 0 & 0 & \cdots & q & -1 \\
\end{bmatrix}
\begin{bmatrix}
f_{00} \\
f_{10} \\
f_{20} \\
\vdots \\
f_{n-2,0} \\
f_{n-1,0} \\
\end{bmatrix}
= \begin{bmatrix}
1 \\
0 \\
0 \\
\vdots \\
0 \\
0 \\
\end{bmatrix}
$$
解得$\begin{bmatrix}
f_{00} & f_{10} & f_{20} & \cdots & f_{n-1,0} \\
\end{bmatrix}^{\mathrm{T}}\oplus [f_{n0}]$即为所求的$f_{i0}$。(实在算不出来了)
% $$
% \begin{bmatrix}
% 1 & 0 & 0 & 0 & 0 & 0 & 0 & 0 & 0 & 0 \\
% q & -1 & p & 0 & 0 & 0 & 0 & 0 & 0 & 0 \\
% 0 & q & -1 & p & 0 & 0 & 0 & 0 & 0 & 0 \\
% 0 & 0 & q & -1 & p & 0 & 0 & 0 & 0 & 0 \\
% 0 & 0 & 0 & q & -1 & 0 & 0 & 0 & 0 & 0 \\
% 0 & 0 & 0 & 0 & q & -1 & p & 0 & 0 & 0 \\
% 0 & 0 & 0 & 0 & 0 & q & -1 & p & 0 & 0 \\
% 0 & 0 & 0 & 0 & 0 & 0 & q & -1 & p & 0 \\
% 0 & 0 & 0 & 0 & 0 & 0 & 0 & q & -1 & p \\
% 0 & 0 & 0 & 0 & 0 & 0 & 0 & 0 & q & -1 \\
% \end{bmatrix}^{-1}
% \begin{bmatrix}
% 1 \\
% 0 \\
% 0 \\
% 0 \\
% 0 \\
% 0 \\
% 0 \\
% 0 \\
% 0 \\
% 0 \\
% \end{bmatrix} = \begin{bmatrix}1\\\frac{q (- 2 p q + 1)}{p^{2} q^{2} - 3 p q + 1}\\\frac{q^{2} (- p q + 1)}{p^{2} q^{2} - 3 p q + 1}\\\frac{q^{3}}{p^{2} q^{2} - 3 p q + 1}\\\frac{q^{4}}{p^{2} q^{2} - 3 p q + 1}\\\frac{q^{5}}{3 p^{2} q^{2} - 4 p q + 1}\\\frac{q^{6} (- 2 p q + 1)}{3 p^{4} q^{4} - 13 p^{3} q^{3} + 16 p^{2} q^{2} - 7 p q + 1}\\- \frac{q^{7}}{3 p^{3} q^{3} - 10 p^{2} q^{2} + 6 p q - 1}\\\frac{q^{8}}{3 p^{4} q^{4} - 13 p^{3} q^{3} + 16 p^{2} q^{2} - 7 p q + 1}\\\frac{q^{9}}{3 p^{4} q^{4} - 13 p^{3} q^{3} + 16 p^{2} q^{2} - 7 p q + 1}\end{bmatrix}
% % = \begin{bmatrix}1\\\frac{q (- 2 p q + 1)}{p^{2} q^{2} - 3 p q + 1}\\\frac{q^{2} (- p q + 1)}{p^{2} q^{2} - 3 p q + 1}\\\frac{q^{3}}{p^{2} q^{2} - 3 p q + 1}\\\frac{q^{4}}{p^{2} q^{2} - 3 p q + 1}\end{bmatrix}
% $$
% $$
% \begin{bmatrix}
% 1 & 0 & 0 \\
% q & -1 & p \\
% 0 & q & -1 \\
% \end{bmatrix}^{-1}
% \begin{bmatrix}
% 1 \\
% 0 \\
% 0 \\
% \end{bmatrix} = \begin{bmatrix}1\\- \frac{q}{p q - 1}\\- \frac{q^{2}}{p q - 1}\end{bmatrix}
% $$
\end{solution}
}
\end{enumerate}
\end{document}